Inequality Proof – How to Prove $\sum\limits_{k=1}^n \frac{a_k}{a_{k+1} +a_{k+2}}\ge \frac{n}{4}$ for $a_k>0$

inequalitysequences-and-seriessummation

For $k=1,2,\dots,n$ and $a_k>0$ let $a_{n+1}=a_1, a_{n+2}= a_2$ I want to prove that $$\sum\limits_{k=1}^n \frac{a_k}{a_{k+1} +a_{k+2}}\ge \frac{n}{4}.$$

Context: I saw this problem on my problem book and after an hour of thinking I couldn't get to anything useful.
I tried to use AM-HM inequality and Cauchy's inequality but this lead to nothing.


Also What values of $a_k$ achieves $\sum\limits_{k=1}^n \frac{a_k}{a_{k+1} +a_{k+2}}= \frac{n}{4}$?

In these type of equation it is usually when $a_k$ is a constant sequence (like Cauchy's inequality when $a_k=b_k$ and AM-GM-HM inequality when $a_k$ is a constant sequence).

Best Answer

This is a classic proof of this weakened Shapiro inequality. Let $x_1$ be the maximum of $a_i,$ $i=1,…,n$. Let $x_2$ be the maximum of the two numbers following $x_1$ in the sequence $\{a_i\}$. Then let $x_3$ be the maximum of the two numbers following $x_2$ and so on. This process will end up back in $x_1$. Then we can say that the LHS of your inequality is greater or equal to

$$\frac{x_1}{2x_2}+ \frac{x_2}{2x_3}+\dots+ \frac{x_m}{2x_1}$$

which is obviously (due to AM-GM) $\geq m/2 \geq n/4$.